LSAT and Law School Admissions Forum

Get expert LSAT preparation and law school admissions advice from PowerScore Test Preparation.

 Administrator
PowerScore Staff
  • PowerScore Staff
  • Posts: 8916
  • Joined: Feb 02, 2011
|
#45695
Complete Question Explanation
(The complete setup for this game can be found here: lsat/viewtopic.php?t=16967)

The correct answer choice is (E)

Answer choice (A) is incorrect because only one student is selected (K and L are both selected here).

Answer choice (B) is incorrect because if F is selected Z must also be selected.

Answer choice (C) is incorrect because if W is selected H must also be selected.

Answer choice (D) is incorrect because U and W cannot be selected together.

Answer choice (E) is the correct answer choice.

Get the most out of your LSAT Prep Plus subscription.

Analyze and track your performance with our Testing and Analytics Package.